Hermiticidad del operador de Dirac en el espacio-tiempo curvo

El Lagrangiano de Dirac en el espacio-tiempo curvo suele estar dado por

L = i Ψ ¯ γ a mi a m ( m + 1 4 ω m b C γ b γ C ) Ψ
De acuerdo con la sección 7.10.3 de "Geometría, topología y física" de Nakahara. este lagrangiano no es hermitiano, pero podemos reescribirlo en forma hermitiana como
L = 1 2 ( L + L ) = i Ψ ¯ γ a mi a m Ψ + 1 4 mi a m ω m b C Ψ ¯ { γ a , γ b C } Ψ
hasta la derivada total. Puedo ver por qué esto es cierto para el término derivado, pero realmente no puedo ver cómo se puede hacer esto para el segundo término. estoy tratando de viajar γ a con γ b C = 1 2 [ γ b , γ C ] de alguna manera, si b , C a entonces esto es fácil { γ a , γ b C } = γ a γ b C y se recupera el segundo término. Pero el problema es si a = b C entonces { γ a , γ b C } = 0 . No veo cómo podemos recuperar un término como mi a m ω m a C γ a γ a γ C en el lagrangiano original (sin suma en a ). Entonces mi pregunta es:

  • ¿Cómo demuestro que los dos Lagrangianos son iguales? (o de manera equivalente, ¿cómo demuestro que L = L hasta la derivada total).
  • Si L y L en realidad no son lo mismo hasta la derivada total (es decir, otro error tipográfico en Nakahara). Entonces, ¿cuál uso como lagrangiano para partículas de espín-1/2 en el espacio-tiempo curvo? He visto a personas usar el primero que también reproduce la ecuación de Dirac en una fuente como Wikipedia http://en.wikipedia.org/wiki/Dirac_equation_in_curved_spacetime . El lagrangiano se usó sin reescribir en forma hermitiana en lugares como la ecuación de "supergravedad" de Freedman (9.1). O en http://arxiv.org/abs/hep-th/0604198 por ejemplo.

estaba esperando que mi b m ω m b C = ω b b C sería 0 por algunas propiedades antisimétricas de ω . Pero esto aparentemente no es cierto, solo los dos últimos índices de ω son antisimétricos. Supongo que hay una razón por la cual estos dos Lagrangianos finalmente darán la misma teoría de campo, pero no veo por qué. ¿Podría alguien por favor explicarme?

Por cierto, disculpe si esta pregunta parece ser muy similar al operador de Dirac en el espacio-tiempo curvo en 2 dimensiones: ¿hermitiano? . Leí esa entrada, pero parece preocuparme por qué deberíamos reescribirla en forma hermitiana, no cómo. Tampoco puedo comentar sobre esa pregunta porque mi reputación es < 50 aparentemente.

Respuestas (1)

Comentarios a la pregunta (v3):

  1. Tenga en cuenta que las matrices gamma se conservan covariantemente 1

    (1) m γ C   =   ω m C b γ b + 1 4 ω m a b [ γ a b , γ C ]   =   0 , m γ v   =   0 ,
    cf. por ejemplo, ref. 1.

  2. Considere la corriente vectorial

    (2) j m   :=   ψ ¯ γ m ψ ,
    dónde ψ es un espinor de Dirac.

  3. La divergencia del campo vectorial (2) es

    (3) d i v mi j   =   mi 1 d m ( mi j m )   =   m j m   = ( 1 )   ψ ¯ ( m γ m + γ m m ) ψ .

  4. Ahora defina la densidad lagrangiana

    (4) L   :=   i mi ψ ¯ γ m m ψ ,
    y la densidad lagrangiana manifiestamente real
    (5) L   :=   1 2 L + C . C .   =   i mi 2 ψ ¯ ( γ m m m γ m ) ψ .

  5. La diferencia es una derivada total del espacio-tiempo.

    (6) L L   =   i mi 2 ψ ¯ ( m γ m + γ m m ) ψ   = ( 3 )   d m ( i mi 2 j m ) ,
    por lo que las ecuaciones de Euler-Lagrange. para L y L son lo mismo.

Referencias:

  1. DZ Freedman y A. Van Proeyen, SUGRA, 2012; ec. (8.37) pág. 180 Ejercicio 8.12.

--

1 Uso la misma notación y convenciones que en mi respuesta Phys.SE aquí .

Muchas gracias. Olvidé por completo el hecho de que m γ v = 0